2020 AMC 12B Problems/Problem 4

Revision as of 21:03, 7 February 2020 by N828335 (talk | contribs) (Created page with "==Problem== The acute angles of a right triangle are <math>a^\circ</math> and <math>b^\circ</math>, where <math>a>b</math> and both <math>a</math> and <math>b</math> are prime...")
(diff) ← Older revision | Latest revision (diff) | Newer revision → (diff)

Problem

The acute angles of a right triangle are $a^\circ$ and $b^\circ$, where $a>b$ and both $a$ and $b$ are prime numbers. What is the least possible value of $b$?


Solution

See Also

2020 AMC 12B (ProblemsAnswer KeyResources)
Preceded by
Problem 3
Followed by
Problem 5
1 2 3 4 5 6 7 8 9 10 11 12 13 14 15 16 17 18 19 20 21 22 23 24 25
All AMC 12 Problems and Solutions

The problems on this page are copyrighted by the Mathematical Association of America's American Mathematics Competitions. AMC logo.png